Difference between revisions of "2002 AMC 10B Problems/Problem 7"

(still needs solution)
 
(Redirected page to 2002 AMC 12B Problems/Problem 4)
 
(2 intermediate revisions by 2 users not shown)
Line 1: Line 1:
== Problem ==
+
#REDIRECT[[2002 AMC 12B Problems/Problem 4]]
 
 
Let <math>n</math> be a positive integer such that <math>\frac {1}{2} + \frac {1}{3} + \frac {1}{7} + \frac {1}{n}</math> is an integer. Which of the following statements is [b]not[/b] true?
 
 
 
<math> \mathrm{(A) \ } 2\text{ divides }n\qquad \mathrm{(B) \ } 3\text{ divides }n\qquad \mathrm{(C) \ } 6\text{ divides }n\qquad \mathrm{(D) \ } 7\text{ divides }n\qquad \mathrm{(E) \ } n>84 </math>
 

Latest revision as of 17:40, 28 July 2011